CR Weaken 1

This topic has expert replies
Master | Next Rank: 500 Posts
Posts: 242
Joined: Fri Aug 01, 2008 8:49 am
Location: Delhi
Thanked: 6 times

CR Weaken 1

by ranjeet75 » Wed May 04, 2011 6:39 am
Farmers in developing countries claim that the United States government, through farm subsidies, is responsible for the artificially low global price of wheat. Because the U.S. government buys whatever wheat American farmers are unable to sell on the open market, American farmers have no incentive to modulate the size of their crops according to the needs of the global market. As a result, American farmers routinely produce more wheat than the global market can absorb and the global price of wheat is kept low. Without these subsidies, the farmers in developing economies claim, American farmers would produce only the amount of wheat that they could sell on the open market and the global price of wheat would rise.

Which of the following, if true, most weakens the claims of the farmers in developing countries regarding the price of wheat?

"¢ Wheat that is not processed for consumption is often used for certain industrial applications.
"¢ Non-governmental buyers of wheat and wheat products are able to predict how much wheat they will need several years in advance.
"¢ The United States government offers similar subsidies to soybean farmers, though the global price of soybeans is significantly higher than that of wheat.
"¢ Other countries, such as Canada and Russia, are likely to produce more wheat if the United States were to reduce its output.
"¢ The price of sorghum, a crop for which the United States government offers no subsidies, is lower than that of wheat.

The OA is [spoiler]D[/spoiler]

My ques is why C is wrong as this is a weaken question and we can take any outside information true.

and in C the soyabean prices are compared with the wheat prices which should not be wrong in Weaken Type ques.

Please clarify

User avatar
Legendary Member
Posts: 1101
Joined: Fri Jan 28, 2011 7:26 am
Thanked: 47 times
Followed by:13 members
GMAT Score:640

by HSPA » Wed May 04, 2011 6:51 am
I would have definitly selected C if the text in double quotes is not there

The United States government offers similar subsidies to soybean farmers, though the global price of soybeans is significantly higher "than that of wheat".
First take: 640 (50M, 27V) - RC needs 300% improvement
Second take: coming soon..
Regards,
HSPA.

Master | Next Rank: 500 Posts
Posts: 242
Joined: Fri Aug 01, 2008 8:49 am
Location: Delhi
Thanked: 6 times

by ranjeet75 » Wed May 04, 2011 7:52 am
I could not understand what role "than that of wheat" play for determining C wrong

User avatar
Legendary Member
Posts: 1101
Joined: Fri Jan 28, 2011 7:26 am
Thanked: 47 times
Followed by:13 members
GMAT Score:640

by HSPA » Wed May 04, 2011 8:38 am
I tried my best

Conclusion : Farm subsidies by govt of united states is responsible for low prices of wheat
Premise 1: Govt will buy excess wheat produced
Premise 2: Farmers' production is more than demand ( price will fall )
Premise 3: No subsidy => less production => high cost and profits

Good weakner: US govt provide similar subsidy for soy beans but their cost havent declined.
=> subsidy has no impact on cost of Soy => subsidy has no impact on wheat.

Does the third world farmers produce soybean???
The demand for soybean is not falling so US soy farmers might have monopoly?

Soy cost lot more than wheat. So a 10% drop in soy might not be significant while the same drop in wheat is significant.
Best business sells its products for 100% margin.
Let wheat production cost = 5 ;sold at = 10 - (10% of 10) = 9 ; profit = 4
Let soy production cost = 20; sold at = 40 - 4 ; 16 is the profit
First take: 640 (50M, 27V) - RC needs 300% improvement
Second take: coming soon..
Regards,
HSPA.

Legendary Member
Posts: 768
Joined: Mon Nov 30, 2009 3:46 am
Thanked: 21 times
Followed by:7 members

by GMATMadeEasy » Thu May 05, 2011 3:28 am
@HSPA:
Good weakner: US govt provide similar subsidy for soy beans but their cost havent declined.
=> subsidy has no impact on cost of Soy => subsidy has no impact on wheat.
You are ASSUMING as if Soya has the same situation as Wheat. We do NOT know that. These are two different prodcuts and you can NOT compare thier prices. What if he gives the same exapple for Almond that is very expensive in all countries.

You find this case as a weakener by thinkig that a similar cause/situation not producing effect will weaken right . You are right in your reasoning but the only difference is that situation is NOT similar here. (Soya and wheat comparison not same)

User avatar
GMAT Instructor
Posts: 905
Joined: Sun Sep 12, 2010 1:38 am
Thanked: 378 times
Followed by:123 members
GMAT Score:760

by Geva@EconomistGMAT » Thu May 05, 2011 3:56 am
GMATMadeEasy wrote:@HSPA:
Good weakner: US govt provide similar subsidy for soy beans but their cost havent declined.
=> subsidy has no impact on cost of Soy => subsidy has no impact on wheat.
You are ASSUMING as if Soya has the same situation as Wheat. We do NOT know that. These are two different prodcuts and you can NOT compare thier prices. What if he gives the same exapple for Almond that is very expensive in all countries.

You find this case as a weakener by thinkig that a similar cause/situation not producing effect will weaken right . You are right in your reasoning but the only difference is that situation is NOT similar here. (Soya and wheat comparison not same)
It's not that C is wrong - C is indeed a weakener - it's just not a very strong weakening case, as we have no idea what else is different between wheat and soy. C is what we call a "second best" - it goes in the right direction, but requires further assumptions or "steps" to weaken the conclusion, whereas D weakens it directly - D says that if the US were to stop doing what it's doing, other countries will step in and replace it, so the same status quo will remain and wheat prices will not grow.
Geva
Senior Instructor
Master GMAT
1-888-780-GMAT
https://www.mastergmat.com

Legendary Member
Posts: 768
Joined: Mon Nov 30, 2009 3:46 am
Thanked: 21 times
Followed by:7 members

by GMATMadeEasy » Thu May 05, 2011 4:45 am
@Geva: The option C compares price of Soya to price of wheat. Isn't this information in itself sufficent to make it OUT of SCOPE ? How could we compare global price of Soya to global price of wheat when these are two different things at least with respect to the current argument.

Author is not comparing decline in the price but absolute prices of two different products . I can definitely be wrong but that was my reasoing for this. Please clarify in case I am missing something hre.

Master | Next Rank: 500 Posts
Posts: 218
Joined: Sat Jul 24, 2010 2:43 pm
Thanked: 5 times

by cyrwr1 » Thu May 05, 2011 7:06 pm
As Geva said, the argument is:

US subsidies increase production--->> causing a drop in price

What weakens this?

If US does not increase production, there were will other reasons that production will still increase.

D is the answer.


Just my thought, after reading the stimulus/stem, do not immediately go to the choices. Think for a sec, what alternatives/reasons the conclusion is weak?